What is the measure of Angle D F E?

Answers

Answer 1

Answer:

Could you please put a picture of the problem?

Step-by-step explanation:

We don't know what you are trying to solve.


Related Questions

HELPPPPPPP PLEASEEEEEEE
Find the solutions of the quadratic equation:

Answers

Answer:

Answer A is correct

Step-by-step explanation:

Let us use completing square method for this.

[tex]x^{2} -5x -10=0\\\\x^{2} -5x=10\\\\x^{2} -5x+\frac{25}{4}=10+ \frac{25}{4}\\\\ (x+\frac{5}{2}) ^{2} =\frac{10*4}{1*4} +\frac{25}{4}\\\\ (x+\frac{5}{2}) ^{2} =\frac{40}{4} +\frac{25}{4}\\\\ (x+\frac{5}{2}) ^{2} =\frac{40+25}{4} \\\\(x+\frac{5}{2}) ^{2} =\frac{65}{4} \\\\(x+\frac{5}{2}) = \±\sqrt{\frac{65}{2} }\\\\(x+\frac{5}{2}) =\±\frac{\sqrt{65}}{4}\\\\x=\±\frac{\sqrt{65}}{4}-\frac{5}{2}[/tex]

∴ [tex]x=-\frac{5}{2} \±\frac{\sqrt{65}}{4}[/tex]

5 Quick algebra 1 questions for 50 points!


Only answer if you know all 5, Tysm! :)

Answers

The equations of the perpendicular lines are: y = 1/2x + 6, y = 15, y = -x - 2, y = 6x + 3 and y = 1/3x - 4

How to determine the equations?

When a linear equation is represented as:

Ax + By = C

The slope (m) is:

m = -A/B

When the linear equation is represented as:

y = mx + c

The slope is m

A line perpendicular to a linear equation that has a slope of m would have a slope of -1/m

Using the above highlights, the equations of the lines are:

6. y = -2x + 5; (2, 7)

The slope is:

m = -2

The perpendicular slope is:

n = 1/2

The equation of the perpendicular line is:

y = n(x - x1) + y1

This gives

y = 1/2(x - 2) + 7

Evaluate

y = 1/2x - 1 + 7

This gives

y = 1/2x + 6

7. y = -5; (11, 15)

The slope is:

m = 0

The perpendicular slope is:

n = 1/0 = undefined

The equation of the perpendicular line is:

y = n(x - x1) + y1

This gives

y = 15

8. Graph ; (-12, 10)

The slope is:

m = (y2 - y1)/(x2 - x1)

Using the points on the graph, we have:

m = (2 - 3)/(3 - 4)

m = 1

The perpendicular slope is:

n = -1

The equation of the perpendicular line is:

y = n(x - x1) + y1

This gives

y = -1(x + 12) + 10

y = -x - 12 + 10

Evaluate

y = -x - 2

9. y = -1/6x + 1; (-2, -9)

The slope is:

m = -1/6

The perpendicular slope is:

n = 6

The equation of the perpendicular line is:

y = n(x - x1) + y1

This gives

y = 6(x + 2) - 9

Evaluate

y = 6x + 12 - 9

This gives

y = 6x + 3

10. 6x + 2y = 14; (12, 0)

The slope is:

m = -6/2

m = -3

The perpendicular slope is:

n = 1/3

The equation of the perpendicular line is:

y = n(x - x1) + y1

This gives

y = 1/3(x - 12) + 0

Evaluate

y = 1/3x - 4

Hence, the equations of the perpendicular lines are: y = 1/2x + 6, y = 15, y = -x - 2, y = 6x + 3 and y = 1/3x - 4

Read more about linear equations at:

https://brainly.com/question/13763238

#SPJ1

What are the zeros of this function?
A. X=2 and x=-4
B. x=0 and x=4
C. X=0 and x=2
D. X=0 and x=-4

Answers

Answer:

The answer is B: x = 0 and x = 4

Step-by-step explanation:

The zeroes of a function are the x-intercepts, or the x values of the points that are on the x-axis. (The x-axis is the numbered horizontal line.)

One of the zeroes passes through the origin, which is (0,0). The other point is right before the number 5 on the x axis, which means that the point is (0, 4). If you look at the x-values of the ordered pairs of the zeroes, that's the answer.

(x, y)

Geometry question I need help with the true or false

Answers

Answer:

See below

Step-by-step explanation:

SAS says the two triangles are congruent

 so   side 6x-4 = 3x+8     then x = 4  

x= 4 true

x = 3 false ( because we just found it = 4)

Pythag theorem says QT

   (6x-4)^2 = QT^2 + 12^2

   ( 6(4)-4)^2 = 12 ^2 + QT^2

       400  -144 = QT^2

       QT = 16   true

At a football match, there were 250 more men than women. The number of children was twice the number of women and the number of men was twice the number of women and children combined. How many people were at the match.​

Answers

Answer:

450 people were at the match

Step-by-step explanation:

So let's make equation :

W = women

M = Men

C = Children

First equation :

M = W + 250

Second equation :

C = 2W

Third equation :

M = 2(W+C)

Let's substitute the second equation into the third :

M = 2(W + 2W)

M = 2W + 4W

M = 6W

Let's substitute this equation into the first one :

6W = W + 250

5W = 250

W = 50

Now we know that that there were 50 women at the match and can substitute this value for the first equation and then the second :

M = 50 + 250

M = 300

This means that there must have been 300 men at the match.  

C = 2(50)

C = 100

This means that there must have been 100 children at the match.

Adding all values together gives us :

50 + 300 + 100  =

450 people were at the match

Hope this helped and have a good day

help me please im in a rush

Answers

Answer:

390 m^2

Step-by-step explanation:

So let's start by finding the surface area of the two triangles on the side. Generally the area of a triangle can be calculated as: [tex]\frac{1}{2}bh[/tex]. but since there is two of them, we can calculate the area of both of them by simply canceling out the 1/2 to a 1. So the area of both triangles are 12 m * 5m as given in the diagram. This gives you 60m^2 area for both triangles on the side

Now let's calculate the rectangle on the top. It has a width and length of 13 and 11 m. So multiply these together to get: 143 m^2

Now let's calculate the rectangle that's on the bottom, it has the dimensions 11 and 12m and multiplying these together gets you: 132 m^2\

Now calculate the rectangle all the way in the back, it has the dimensions 5 and 11m, multiplying these together gets you 55 m^2.

Now add all these together: 60m^2 + 143m^2 + 132m^2 + 55m^2 = 390m^2

what type of polynomial is -3

Answers

Answer:

a monomial, a constant

Step-by-step explanation:

It has only 1 term, so it's a monomial.

Also, the term is a constant.

It is in fact a monomial

Question 1 of 5
Select the correct answer.
Which function has a domain of (-♾, ♾)and a range of (-♾, 4)?

PLEASE HELP

A: f(x) = -x^2 + 4

B: f(x) = 2^x +4

C: f(x) = -4x

D: f(x) = x + 4

Answers

Answer: A

Step-by-step explanation:

A) Correct. [tex]x^2 \geq 0 \implies -x^2 \leq 0[/tex]

B) Wrong. [tex]2^x > 0[/tex], so the range is [tex](4, \infty)[/tex]

C) Wrong. Linear functions have a range of all real numbers.

D) Wrong. Same logic as C.

Please help me to find the value of x and y . As fast as possible....​

Answers

Answer:

Step-by-step explanation:

Answer:

y = 40°

z = 140°

x = 100°

Information:

(i) Sum of interior angles of a triangle sum ups to 180°

(ii) On a straight line, the angles sum up to 180°

(iii) One exterior angle is equal to two opposite interior angles.

Solve for z

Here the exterior angle theorem applies.

∠z = 120° + 20°

∠z = 140°

Solve for y

Find the angle C. Here angles lie on a straight line.

∠? + 120° = 180°

∠? = 180° - 120° = 60°

80°, 60° and y are interior angles of a triangle.

y + 80°+ 60° = 180°

y = 180° - 140°

y = 40°

Solve for x

∠? = 40° (vertically opposite angle)

Now,

y + x + 40° = 180°

40° + x + 40° = 180°

x = 100°

Which graph represents an exponential function?

Answers

The first graph is the exponential function.

What is exponential function?

The exponential function serves as the  positive-valued function with respect to real variable. and  the values denoted as X which is exponential is  all real numbers.

Analyzing the first graph, moving down the graph, we can see that  x value increases.

Learn more about exponential function at:

https://brainly.com/question/12394974

#SPJ1

Answer:

First Graph

Step-by-step explanation:

Edge

What is the solution to the system of equations?

y = 2/3x + 3
x =-2

[-2, -15/2 ]
[-2, 5/3]
(-2, 11/6
(-2, 13/3)

Answers

Answer:

(- 2, [tex]\frac{5}{3}[/tex] )

Step-by-step explanation:

y = [tex]\frac{2}{3}[/tex] x + 3 → (1)

x = - 2

substitute x = - 2 into (1)

y = [tex]\frac{2}{3}[/tex] × - 2 + 3 = - [tex]\frac{4}{3}[/tex] + [tex]\frac{9}{3}[/tex] = [tex]\frac{5}{3}[/tex]

solution is (- 2, [tex]\frac{5}{3}[/tex] )

"I am thinking of a number. If you
multiply my number by 4 and then add 3
times my number, you will get 175. What is
my number?"

Answers

Answer:

y =25

Step-by-step explanation:

y×4+3×y=175 4y+3y=175 7y=175 divide by 7 both side with make value of y y=25. this is my answer

which inequality represents all values of x for which the quotient below is defined? sqrt 7x^2 / sqrt3x

Answers

Answer:

The Answer is x>0

Find dy/dx by implicit differentiation.

(sin pix + cos piy)^8 = 17

dy/dx =


please help quickly

Answers

dy/dx by implicit differentiation is cos(πx)/sin(πy)

How to find dy/dx by implicit differentiation?

Since we have the equation

(sin(πx) + cos(πy)⁸ = 17, to find dy/dx, we differentiate implicitly.

So, [(sin(πx) + cos(πy)⁸ = 17]

d[(sin(πx) + cos(πy)⁸]/dx = d17/dx

d[(sin(πx) + cos(πy)⁸]/dx = 0

Let sin(πx) + cos(πy) = u

So, du⁸/dx = 0

du⁸/du × du/dx = 0

Since,

du⁸/du = 8u⁷ and du/dx = d[sin(πx) + cos(πy)]/dx

= dsin(πx)/dx + dcos(πy)/dx

= dsin(πx)/dx + (dcos(πy)/dy × dy/dx)

= πcos(πx) - πsin(πy) × dy/dx

So, du⁸/dx = 0

du⁸/du × du/dx = 0

8u⁷ × [ πcos(πx) - πsin(πy) × dy/dx] = 0

8[(sin(πx) + cos(πy)]⁷ ×  (πcos(πx) - πsin(πy) × dy/dx) = 0

Since 8[(sin(πx) + cos(πy)]⁷ ≠ 0

(πcos(πx) - πsin(πy) × dy/dx) = 0

πcos(πx) = πsin(πy) × dy/dx

dy/dx = πcos(πx)/πsin(πy)

dy/dx = cos(πx)/sin(πy)

So, dy/dx by implicit differentiation is cos(πx)/sin(πy)

Learn more about implicit differentiation here:

https://brainly.com/question/25081524

#SPJ1

for what value of k, the line joining 3x-ky+7=0 is perpendicular to the line joining (4 ,3) and ( 5, -3).

Answers

Answer:

k = 18

=========

Given

Line 13x - ky + 7 = 0

Line 2Passing through the points (4, 3) and (5, - 3)

To find

The value of k, if the lines are perpendicular

Solution

We know the perpendicular lines have opposite reciprocal slopes, that is the product of their slopes is - 1.

Find the slope of line 1 by converting the equation into slope-intercept from standard form:

Info:

standard form is ⇒ ax + by + c = 0, slope - intercept form is ⇒ y = mx + b, where m is the slope

3x - ky + 7 = 0ky = 3x + 7y = (3/k)x + 7/k

Its slope is 3/k.

Find the slope of line 2, using the slope formula:

m = (y₂ - y₁)/(x₂ - x₁) = (-3 - 3)/(5 - 4) = - 6/1 = - 6

We have both the slopes now. Find their product:

(3/k)*(- 6) = - 1- 18/k = - 1k = 18

So when k is 18, the lines are perpendicular.

Please help me figure out where they all belong to

Answers

The values of the trigonometric functions are

[tex]sin \theta = \frac{12}{13}[/tex]

[tex]cos \theta = \frac{5}{13}[/tex]

[tex]tan \theta = \frac{12}{5}[/tex]

 [tex]cot\theta = \frac{5}{12}[/tex]

[tex]csc\theta = \frac{13}{12}[/tex]

Trigonometric functions

From the question, we are to determine the values of the given trigonometric functions

From the given information,

[tex]sec \theta =\frac{13}{5}[/tex]

∴ [tex]\frac{1}{cos \theta} =\frac{13}{5}[/tex]

[tex]cos \theta = \frac{5}{13}[/tex]

Thus,

Adjacent = 5

Hypotenuse = 13

Opposite = ?

Using the Pythagorean theorem

|Opp|² = |Hyp|² - |Adj|²

|Opp|² = 13² - 5²

|Opp|² = 169 - 25

|Opp|² = 144

|Opp| = √144

|Opp| = 12

∴ Opposite = 12

Thus,

By using SOH CAH TOA

[tex]sin \theta = \frac{12}{13}[/tex]

[tex]tan \theta = \frac{12}{5}[/tex]

[tex]cot\theta = \frac{1}{tan\theta}[/tex]

∴ [tex]cot\theta = \frac{5}{12}[/tex]

[tex]csc\theta = \frac{1}{sin\theta}[/tex]

∴ [tex]csc\theta = \frac{13}{12}[/tex]

Hence, the values of the trigonometric functions are

[tex]sin \theta = \frac{12}{13}[/tex]

[tex]cos \theta = \frac{5}{13}[/tex]

[tex]tan \theta = \frac{12}{5}[/tex]

 [tex]cot\theta = \frac{5}{12}[/tex]

[tex]csc\theta = \frac{13}{12}[/tex]

Learn more on Trigonometric functions here: https://brainly.com/question/13276558

#SPJ1

Sam runs 36 km in 2.5 hours, how many km does he run per hour

Answers

Answer:

Sarah runs at 14.4km/h

Step-by-step explanation:

36km = 2.5 hours

/ = per or divide

36km/2.5hours = 36 divided by 2.5 = 14.4km/h

He runs 14.4 km per hour.

31. Lefties Find the sample size needed to estimate the percentage of California residents who are left-handed. Use a margin of error of three percentage points, and use a confidence level of 99%. a. Assume that pn and qn are unknown. b. Assume that based on prior studies, about 10% of Californians are left-handed. c. How do the results from parts (a) and (b) change if the entire United States is used instead of California

Answers

a. The required sample size when pn and qn are unknown then n = 1844

b. The required sample size when about 10% of Californians are left-handed is n = 664

c. There would have been no effect on parts a and b if entire US has been chosen

According to statement

a. No estimate of proportion is given so we will assume:  

{p} = {q} = 0.5

For 99% confidence, z = 2.576

E = 0.03  

Hence, Required sample size  

n = (0.5)*(0.5)*(2.576/0.03)^2  

n = 1844

b. {p} = 0.10

{q} = 1 - {p}

{q} = 1 - 0.10

{q} = 0.90  

For 99% confidence, z = 2.576

E = 0.03

Hence,  Required sample size  n = (0.1)*(0.9)*(2.576/0.03)^2  = 664

c. There would have been no effect on parts a and b if entire US has been chosen because minimum sample size required is not dependent on population size.

Learn more about PERCENTAGE here https://brainly.com/question/24304697

#SPJ4


When 1,2504 is written in its simplest radical form, which value remains under the radical?
O 10
O 6
O 5
02

Answers

The value that remains under the radical is 2

How to determine the value under the radical?

The correct expression in the question is:

1250^4

This can be rewritten as:

[tex]\sqrt[4]{1250}[/tex]

Express 1250 as product

[tex]\sqrt[4]{(2 * 5^4)}[/tex]

Expand the expression

[tex]\sqrt[4]{2} * \sqrt[4]{5^4}[/tex]

Simplify

[tex]\sqrt[4]{2} * 5[/tex]

Evaluate the product

[tex]5\sqrt[4]{2}[/tex]

Hence, the value that remains under the radical is 2

Read more about radical expressions at:

https://brainly.com/question/3008670

#SPJ1

Question 17 of 25
how many solutions does the following system of equations have?
y=5/2x+2
2y=5x+4
a. zero
b. one
c.infinitely many
d. two

Answers

Answer:

c

Step-by-step explanation:

y = [tex]\frac{5}{2}[/tex] x + 2 → (1)

2y = 5x + 4 ( divide through by 2 )

y = [tex]\frac{5}{2}[/tex] x + 2 → (2)

the 2 equations are the same and will have infinitely many solutions

What is the answer to the question below

Answers

The distance between the two given points coordinates is; Option C: √85

How to find the distance between two coordinates?

Formula for the distance between two coordinates is;

d = √((x₂ - x₁)² + (y₂ - y₁)²)

We are given the coordinates as;

(-2, 3) and (5, -3). Thus;

d = √((-2 - 5)² + (-3 - 3)²)

d = √(49 + 36)

d = √85

Read more about distance between two coordinates at; https://brainly.com/question/7243416

#SPJ1

Helppp meeeeee pleasseeeeeeeeeee

Answers

Answer:

Khan Academy.

Step-by-step explanation:

You can go on khan academy and search up "percentages and numbers." The videos should immediately pop up. there are also lessons if you don't want to risk getting your problem wrong. If you are still confused and do not understand please just comment and I will respond.

On a coordinate plane, 2 trapezoids are shown. Trapezoid M O N P has points (negative 5, 4), (negative 2, 5), (negative 2, 2), and (negative 5, 3). Trapezoid C D E F has points (3, negative 5), (4, negative 5), (5, negative 2), (2, negative 2).

Trapezoid CDEF was reflected across the x-axis followed by a 90° rotation about the origin to create the other trapezoid shown on the graph. Which congruency statement applies to the trapezoids?
CDEF ≅ NPOM
CDEF ≅ MNPO
CDEF ≅ NMOP
CDEF ≅ MOPN

Answers

A trapezoid is a quadrilateral which is having a pair of opposite sides as parallel and the length of the parallel sides is not equal. The correct option is C.

What is a Trapezoid?

A trapezoid is a quadrilateral which is having a pair of opposite sides as parallel and the length of the parallel sides is not equal.

Trapezoid CDEF was reflected across the x-axis followed by a 90° rotation about the origin to create the other trapezoid shown on the graph. The congruency statement that applies to the trapezoids is CDEF ≅ NMOP.

Hence, the correct option is C.

Learn more about Trapezoid:

https://brainly.com/question/8643562

#SPJ1


Find the Greatest Common Factor of Two or More Expressions
In the following exercises, find the greatest common factor.
3. 72,162

Answers

Answer:

The greatest common factor of 72 and 162 is 18.

Step-by-step explanation:

In my opinion, one of the easier ways to find the greatest common factor of two numbers is to get the prime factors of both and multiply the common ones. The prime factorization of 72 is 2*2*2*3*3, and the prime factorization of 162 is 2*3*3*3*3 (you can make a factor tree for both of these numbers to verify this). The common primes factors for both of these numbers are 2, 3, and 3 (both 72 and 162 have one 2 and at least two 3s). 2*3*3 is 18, which is the greatest common factor.

2. Try It #2 The population of a small town increased from 1,442 to 1,868 between 2009 and 2012 . Find the change of population per year if we assume the change was constant from 2009 to 2012 .

Answers

Answer:

If it is assumed that the change was constant from 2009 and 2012 , then change of population is 142 .

Step-by-step explanation:

In the question, it is given that the population of a small town increased from 1442 to 1868 between 2009 and 2012 .

It is asked to find the change of population if it is assumed that the change was constant from 2009 and 2012 .

Step 1 of 2

Population in 2012 is given to be 1868 and

Population in 2009 is given to be 1442 .

Duration of year is given by 2012-2009 which is 3 years.

Change in population is given by 1868-1442 which is 426 .

Step 2 of 2

Change in population per year is given by dividing the change in population and the duration of years it took to happen the change.

Hence, change in population per year is

[tex]$\frac{426}{3}=142$[/tex]

Hence, the population was changed by 142 per year from 2009 to 2012 .

Which of the following is a valid exclusion for the algebraic fraction 8ab^2x/4a^2b - 8ab^2
A. a=/ 0, b =/ 0, a = / b
B. a = / 0, b =/ 0
C. a =/ 0, b =/ 0, a = / 2b
D. b = / 0, a = / b

Answers

The valid exclusion of the algebraic fraction is (c) a =0, b =0, a =2b

How to determine the valid exclusion?

The expression is given as:

8ab^2x/4a^2b - 8ab^2

Set the denominator to 0

4a^2b - 8ab^2 = 0

Divide through by 4ab

a - 2b = 0

Add 2b to both sides

a = 2b

Hence, the valid exclusion of the algebraic fraction is (c) a =0, b =0, a =2b

Read more about algebraic fraction at:

https://brainly.com/question/4344214

#SPJ1

How many units are in the sum of the lengths of the three altitudes in a triangle with sides $7,$ $24,$ and $25$

Answers

56 units are in the sum of the lengths of the three altitudes in a triangle with sides $7,$ $24,$ and $25$

Properties of triangles are:

A triangle has three sides and three angles.

The sum of the angles of a triangle is always 180 degrees.

The exterior angles of a triangle always add up to 360 degrees.

The sum of consecutive interior and exterior angle is supplementary.

How to solve?

Given:

Height: ha = 7

Height: hb = 24

Height: hc = 25

Sum to sides is:

25 +7 + 24 =56 units.

Learn more about Triangles:

https://brainly.com/question/4313883

#SPJ4

From the diagram below, given the side lengths marked, and if we know that < C is congruent to < E, we can say that ___.



Select one:

a.
the two triangles are similar by SAS


b.
the two triangles are not similar


c.
the two triangles are congruent


d.
the two triangles are similar by AA

Answers

Since we know that <C is congruent to <E, we can say that: A. the two triangles are similar by SAS.

The properties of similar triangles.

In Geometry, two triangles are similar when the ratio of their corresponding sides are equal in magnitude and their corresponding angles are congruent.

Ratio = BC/AC = 6/3 = 2.

Ratio = FE/DE = 4/2 = 2.

Thus, the ratio of their corresponding sides are equal in magnitude i.e BC/AC ≡ FE/DE.

Since we know that <C is congruent to <E, we can say that the two triangles are similar by side, angle, side (SAS) criterion.

Read more on congruency here: brainly.com/question/11844452

#SPJ1

Two systems of equations are shown:

System A System B

6x + y = 2 2x − 3y = −10

−x − y = −3 −x − y = −3


Which of the following statements is correct about the two systems of equations?


The value of x for System B will be 4 less than the value of x for System A because the coefficient of x in the first equation of System B is 4 less than the coefficient of x in the first equation of System A.


They will have the same solution because the first equations of both the systems have the same graph.


They will have the same solution because the first equation of System B is obtained by adding the first equation of System A to 4 times the second equation of System A.


The value of x for System A will be equal to the value of y for System B because the first equation of System B is obtained by adding −4 to the first equation of System A and the second equations are identical.

Answers

The correct statement is that the value of x for System A will be equal to the value of y for System B because the first equation of System B is obtained by adding −4 to the first equation of System A and the second equations are identical. Option D

How to determine the right statement

6x + y = 2 2x − 3y = −10    System A

-x − y = −3 −x − y = −3 System B

Let;s solve for x and y in system A

6x + y = 2

Make 'y' the subject

y = 2-6x

Substitute in the other equation

-x -y = -3

-x - (2-6x) = -3

-x -2+6x = -3

Collect like terms

5x = -3+2

x = -1/5

Substitute in y = 2-6x to find 'y'

y = 2- 6(-1/5)

y = 2+ 6/5

y = [tex]\frac{10+ 6}{5}[/tex]

y = 16/5

For system B

-x-y = -3

Make y subject, we have

-x + 3 = y

y = -x + 3

Substitute in the other equation, we have

2x − 3y = −10

2x - 3(-x+3) = -10

2x + 3x -9 = -10

Collect like terms

5x -9 = -10

5x = -10 + 9

x = 1/5

Substitute into y = -x + 3 to find 'y'

y = -(1) + 3

y = 2

Thus, the correct statement is that the value of x for System A will be equal to the value of y for System B because the first equation of System B is obtained by adding −4 to the first equation of System A and the second equations are identical. Option D

Learn more about simultaneous equations here:

https://brainly.com/question/16863577

#SPJ1

Given the domain {-2, 1, 5}, what is the range for the relation 4x + y = 3?

Answers

Answer:

Step-by-step explanation:

Domain is the set of all possible inputs (x) for the function f(x)

The range is the set of all possible values for that domain

This function is 4x + y = 3 which can be re-written as y = 3-4x

Plug in each of the domain values and you will get the range

For -2 : y = 3-4(-2) = 3 + 8 = 11

For 1:  y = 3-4(1) = -1

For 5: y = 3-4(5) = -17

So range is {-17, -1, 11}  

Other Questions
The plane 15x 12y + 20z = 60 is represented by which graph below? Notice the z-axis. What does 64. 75 inch height mean? Two genes are 14 map units apart on the X chromosome. What percent of the father's gametes will be recombinant Which pipe for water is best for construction? This occurs when microorganisms are transferred from one infected person directly into another person. In a hospital maternity ward there are only five babies. Baby 1 is heavier, with red hair. Baby 2 is male and thin, with the same colour hair as Baby 3, who is blonde and wears a bonnet. Baby 4 is one of the three female babies and Baby 5 wears sunglasses and is heavier. All babies wear bonnets and at least one of the females wears sunglasses.A thin, red-haired baby wearing a bonnet is the oldest of the group. Who can it be? Identify the most efficient way to protect biodiversity. Explain your reasoning. I'll mark brainest for anyone that can answer this (definition of monocotyledon plant) Mortgage payments and student loans are examples of... A) assets B) credit Eliminate C) collateral D) income Suppose the theoretical probability of winning a prize at a carnival game is 0.18. Based on this probability how many more customers would you have expected to win a prize Comparing two unlike things using like or as is a(n) _____?similemetaphoridiomalliteration Help me with the questions please Directions: Read the paragraphs below. Then choosethe best main idea from the multiple choice options. Highlight or underline one supporting detail in the text. An example has been done for you. URGENT!! WORTH 27 POINTSJace has consumed 1,500 calories so far today. He has also burned off 400 calories at the gym. He would like to keep his daily calorie total to 1,900 calories per day. How many calories does he have left to consume for the day? Is 1,100 a viable solution to this problem?a. yes, 1,100 is less than 1,500.b. no, 1,100 is more than the 400 he burned off at the gym.c. no, 1,100 will cause him to exceed 1,900.d. yes, 1,100 is less than 1,900. A boat moves through the water with two forces acting on it. One is a 1,800-N forward push by the water on the propeller, and the other is a 1,200-N resistive force due to the water around the bow.(a) What is the acceleration of the 1,400-kg boat?0.25 m/s 2(b) If it starts from rest, how far will the boat move in 20.0 s? m(c) What will its velocity be at the end of that time? m/s Dada art could be considered asA. dreamlike.B. absurd.C. perceptual. D. naturalistic. Conteste la siguiente pregunta.Cundo tienes que entregar tu tarea de geografa?O A. Tengo tres tareas.OB. S, quiero estudiar geografa.OC. Tengo que entregarla el prximo jueves.D. Tengo que entregrsela a mi profesora. When you purchase an item from Globus Corp., they place the item in a paper bag with handles and vertical yellow, blue, and white stripes. Even without seeing the words "Globus Corp." on the bag, many people recognize that the purchase is from Globus. The bag's coloring and design is considered its ________. Group of answer choices trademark collective mark certification mark service mark trade dress A perfectly competitive firm typically ________ its marginal revenue with each additional item sold, and a monopolist ________ its marginal revenue as the quantity of sales increases. Which of the following does NOT demonstrate the law of conservation of matter? check all that apply 3) Do you think that menu layout influences your decision to order one thing over another?